Time limit: 0
Quiz Summary
0 of 22 Questions completed
Questions:
Information
You have already completed the quiz before. Hence you can not start it again.
Quiz is loading…
You must sign in or sign up to start the quiz.
You must first complete the following:
Results
Quiz complete. Results are being recorded.
Results
0 of 22 Questions answered correctly
Your time:
Time has elapsed
You have reached 0 of 0 point(s), (0)
Earned Point(s): 0 of 0, (0)
0 Essay(s) Pending (Possible Point(s): 0)
Categories
- Not categorized 0%
- 1
- 2
- 3
- 4
- 5
- 6
- 7
- 8
- 9
- 10
- 11
- 12
- 13
- 14
- 15
- 16
- 17
- 18
- 19
- 20
- 21
- 22
- Current
- Review
- Answered
- Correct
- Incorrect
-
Question 1 of 22
1. Question
A 66-year-old woman comes to the office due to 2 months of worsening fatigue and dyspnea with moderate exertion. The patient has a history of breast cancer treated with surgery and combination chemotherapy 6 years ago. She has no other medical conditions. Vital signs are within normal limits. Physical examination reveals normal jugular venous pressure, clear lungs, and normal heart sounds; there is no hepatosplenomegaly, lymphadenopathy, or extremity edema. Laboratory results are as follows:
Hemoglobin
7.2 g/dL
Mean corpuscular volume
108 µm3
Reticulocytes
1%
Platelets
90,000/mm3
Leukocytes
3,800/mm3
Peripheral blood smear shows oval macrocytic red cells and hyposegmented neutrophils. Bone marrow biopsy of this patient is most likely to reveal which of the following?
CorrectIncorrect -
Question 2 of 22
2. Question
A 73-year-old woman is brought to the emergency department due to headache and confusion. Approximately an hour ago, the patient fell while walking down the stairs and hit her head on the banister. She did not lose consciousness but has had worsening headache, nausea, and vomiting. She has also become incoherent. The patient has a history of persistent atrial fibrillation, hypertension, and end-stage renal disease. Her current medications include warfarin, amlodipine, metoprolol, and calcium acetate. She received maintenance hemodialysis earlier today. Blood pressure is 160/100 mm Hg and pulse is 60/min. The patient is lethargic and oriented to person but not to place or time. Cardiopulmonary auscultation reveals clear lung fields and an irregular heart rate. Muscle strength is equal on both sides. CT scan of the head without contrast reveals a right-sided subdural hematoma with mass effect. Hemoglobin is 11.6 g/dL and INR is 2.1. Surgical hematoma evacuation is planned. Which of the following is the best choice for rapid reversal of anticoagulation in this patient?
CorrectIncorrect -
Question 3 of 22
3. Question
The following vignette applies to the next 2 items
A previously healthy 30-year-old woman is admitted to the hospital. She complains of a rash over her ankles that started 2 weeks ago and slowly spread up to her knees. The patient has no other medical problems and takes no regular medications. Her vital signs are within normal limits. Physical examination shows fine petechiae and purpura around her ankles and lower legs. There is no lymphadenopathy or hepatosplenomegaly. Laboratory results are as follows:
Hemoglobin
14.4 g/dL
Mean corpuscular volume
90 fL
Platelets
16,000/µL
Leukocytes
8,000/µL
Segmented neutrophils
65%
Eosinophils
3%
Lymphocytes
25%
Monocytes
7%
Creatinine
0.7 mg/dL
Prothrombin time
15 sec
Activated partial thromboplastin time
38 sec
Plasma fibrinogen
300 mg/dL (150-350 mg/dL)
Lactate dehydrogenase
72 U/L (60-100 U/L)
D-dimer, plasma
0.22 µg/mL (<0.5 µg/mL)
Peripheral blood smear shows thrombocytopenia. After reviewing the laboratory results, the admitting resident orders 6 units of random donor platelet transfusion. The following day, the platelet count is 10,000/µL.
Item 1 of 2
Which of the following is the most likely cause of the drop in the platelet count?
CorrectIncorrect -
Question 4 of 22
4. Question
Item 2 of 2
Which of the following is the most appropriate next step in management of this patient?
CorrectIncorrect -
Question 5 of 22
5. Question
A 66-year-old man comes to the office due to several weeks of back and lower extremity pain. The pain feels dull and achy and is worsened by movement, causing him to have difficulty bearing weight and ambulating. He has had no bladder or bowel dysfunction, extremity weakness, or numbness. The patient takes metformin for type 2 diabetes mellitus. He drinks 2 alcoholic beverages daily. Physical examination shows no gross abnormality or deformity of the back or extremities. There is no swelling or redness of the legs, and neurological examination is normal. A radiograph of the right lower extremity is shown in the exhibit. Laboratory results are as follows:
Hemoglobin
9.6 g/dL
Mean corpuscular volume
92/µm3
Creatinine
1.6 mg/dL
Calcium
11.0 mg/dL
Glucose
220 mg/dL
Total protein
8.8 g/dL
Albumin
3.8 g/dL
Which of the following is the most likely explanation for this patient’s bone changes?
CorrectIncorrect -
Question 6 of 22
6. Question
A 68-year-old woman comes to the office for follow-up of her anticoagulation therapy. Her medical problems include chronic atrial fibrillation, hypertension, congestive heart failure (CHF) NYHA II, and coronary artery disease. Her medications include daily warfarin 6 mg, lisinopril, furosemide, and carvedilol. She started taking an over-the-counter supplement 2 weeks ago to help increase her energy level but cannot recall the name. She denies bleeding episodes. Echocardiogram performed 2 months ago shows a dilated left atrium and moderately depressed left ventricular ejection fraction. Laboratory evaluation done early in the morning shows an international normalized ratio (INR) of 4.3 with a prothrombin time of 46 sec. Which of the following is the most appropriate next step in management?
CorrectIncorrect -
Question 7 of 22
7. Question
A 65-year-old Caucasian female presents to the emergency department with progressive shortness of breath on minimal exertion and fatigue. She was diagnosed with myelodysplasia two years ago, and has been receiving supportive therapy with frequent RBC transfusions. Her last transfusion was 3 months ago. She experienced an episode of severe pneumonia one year ago that required hospitalization and IV antibiotic therapy. She is taking no medications currently and has no known allergies. Her blood pressure is 120/70 mmHg and heart rate is 95/min. Physical examination reveals pallor. Systolic murmur with intensity of II/VI is heard over the cardiac apex. Laboratory findings are significant for hematocrit of 24% and hemoglobin level of 5.7 mg/dL. You consider RBC transfusion in this patient. Blood grouping and cross-matching are done, but the blood bank is unable to find suitable blood. This is the first time such an incompatibility has occurred. What is the most likely reason for this incompatibility?
CorrectIncorrect -
Question 8 of 22
8. Question
A 30-year-old woman comes to the emergency department with sudden-onset abdominal pain and ascites. Laboratory studies show anemia, reticulocytosis, leukopenia, and thrombocytopenia. Flow cytometry of the patient’s peripheral blood cells using the appropriate monoclonal antibodies shows CD55 and CD59 deficiency. CT scan of the abdomen shows hepatic vein thrombosis. Which of the following is the most likely cause of this patient’s anemia?
CorrectIncorrect -
Question 9 of 22
9. Question
A 54-year old Caucasian man comes to the office complaining of fatigue, poor exercise tolerance, and an unexplained weight loss over the last 6 months. He does not use tobacco, alcohol, or drugs. His family history is noncontributory. He takes no medications. His vital signs are within normal limits. Examination reveals mild splenomegaly but no lymphadenopathy. Laboratory tests show the following:
Hb 9.4 g/dL MCV 84fl Platelet count 110,000/mm3 Leukocyte count 52,000/mm3 Segmented neutrophils 65% Bands 8% Metamyelocytes 6% Blasts 3% Basophils 8% Lymphocytes 11% Monocytes 9% Cytogenetic testing reveals a bcr/abl translocation. Which of the following is the best initial treatment option for this patient?
CorrectIncorrect -
Question 10 of 22
10. Question
A 64-year-old man with diffuse large B-cell lymphoma is hospitalized for chemotherapy initiation. The patient has extensive bulky lymphadenopathy. Medical history also includes hypertension and stage 3 chronic kidney disease. Treatment with cyclophosphamide, doxorubicin, vincristine, and prednisone plus rituximab is begun. The patient is also given intravenous hydration and rasburicase. Four days later, his serum creatinine is acutely elevated and his urine output is decreased. Vital signs are normal. Cardiopulmonary examination is normal. The abdomen is soft with no distension or tenderness. Laboratory studies are as follows:
Complete blood count
Hemoglobin
12.2 g/dL
Platelets
210,000/mm3
Leukocytes
9,000/mm3
Serum chemistry
Sodium
140 mEq/L
Potassium
5.2 mEq/L
Bicarbonate
22 mEq/L
Blood urea nitrogen
40 mg/dL
Creatinine
2.3 mg/dL (baseline: 1.4)
Calcium
6.5 mg/dL
Phosphorus
11.1 mg/dL
Uric acid
5.7 mg/dL
Urinalysis shows no cells or casts. Which of the following is the most likely cause of this patient’s acute kidney injury?
CorrectIncorrect -
Question 11 of 22
11. Question
A 62-year-old Caucasian man comes to the office with complaints of a dull, non-throbbing headache, hearing loss on the left side, and persistent tinnitus. His symptoms have been getting progressively worse for the last two weeks. He has a history of small cell cancer of the lung, which was treated with a combination of chemotherapy and radiotherapy four months ago. His lung mass has reduced considerably in size after its treatment. A contrast-enhanced magnetic resonance imaging scan shows a 1 x 2 cm circumscribed mass in the left cerebellopontine angle compressing the eighth cranial nerve. There is another small 0.5 x 1 cm mass in the left frontal lobe cortex with significant edema surrounding the lesion. Which of the following is the most likely cause of this patient’s symptoms?
CorrectIncorrect -
Question 12 of 22
12. Question
A 35-year-old woman comes to the office for evaluation of fatigue and exertional dyspnea. She has a history of systemic lupus erythematosus and is noncompliant with therapy. The patient takes naproxen as needed for joint pains. Vital signs are within normal limits. Examination shows mild pallor. The remainder of the examination shows no abnormalities. Laboratory results are as follows:
Hemoglobin
8.6 g/dL
Mean corpuscular volume
80 µm3
Creatinine
0.8 mg/dL
Iron, serum
40 µg/dL
Total iron-binding capacity
180 µg/dL
(normal: 250-460 µg/dL)
Lactate dehydrogenase
72 U/L
Which of the following is the most likely underlying cause of her current condition?
CorrectIncorrect -
Question 13 of 22
13. Question
A 29-year-old woman with chronic anemia comes to the emergency department with fatigue and progressive dyspnea. Physical examination shows pallor but no jaundice. Lungs are clear on auscultation and abdominal examination is benign. Laboratory results are as follows:
Hemoglobin 6.2 g/dL Mean corpuscular volume 79 fL Platelets 180,000/µL Leukocytes 7,500/µL Packed red blood cell transfusion is started. Thirty minutes later, the patient complains of severe bilateral flank pain. She has received multiple blood transfusions in the past, but all were uncomplicated. Her temperature is 38.3 C (101 F), blood pressure is 100/70 mm Hg, pulse is 120/min, and respirations are 16/min. Pulse oximetry is 97% on room air. Urine is dark. Which of the following is the best next step in management of this patient?
CorrectIncorrect -
Question 14 of 22
14. Question
A 55-year-old woman is being evaluated for persistent nonproductive cough. She also has had worsening exercise intolerance and fatigue for the past 4 weeks. Physical examination shows conjunctival pallor. Laboratory results are as follows:
Complete blood count
Hemoglobin
6.2 g/dL
Mean corpuscular volume
90 µm3
Reticulocytes
0.1%
Platelets
280,000/mm3
Leukocytes
6,700/mm3
Serum chemistry and kidney function test results are normal. Iron studies and serum vitamin B12 and folic acid levels are within normal limits. Chest imaging reveals an anterior mediastinal mass. Which of the following is the most likely cause of this patient’s hematologic disorder?
CorrectIncorrect -
Question 15 of 22
15. Question
A 54-year-old woman with stage 4 chronic kidney disease due to polycystic kidney disease undergoes elective arteriovenous fistula creation. Excessive bleeding is noted during the surgery, and postoperatively, the patient experiences prolonged oozing that is persistent despite the application of pressure. The patient says she has had recent episodes of easy bruising and epistaxis but recalls no excessive bleeding after cesarean delivery 20 years ago. Medical history also includes hypertension. Vital signs are within normal limits. Preoperative laboratory results are as follows:
Complete blood count
Hemoglobin
9.5 g/dL
Mean corpuscular volume
86 µm3
Platelets
200,000/mm3
Leukocytes
8,000/mm3
Serum chemistry
Blood urea nitrogen
50 mg/dL
Creatinine
4.4 mg/dL
Coagulation studies
PT
12 sec
Activated PTT
23 sec
Which of the following is the best next step in management of this patient’s bleeding?
CorrectIncorrect -
Question 16 of 22
16. Question
A 42-year-old Caucasian woman comes to the emergency department because of right-sided neck pain, persistent cough, and dyspnea for the past three days. Her past medical problems include hypertension and obesity. She takes enalapril and hydrochlorothiazide. She has smoked one pack of cigarettes daily for the past 25 years. She does not drink alcohol. She works as an executive secretary. Her temperature is 37.8C (100 F), blood pressure is 130/80 mm Hg, pulse is 92/min and respirations are 18/min. Examination shows decreased breath sounds in the upper part of the right lung. There is moderate swelling and erythema of the neck, moderate right jugular vein dilatation, and increased venous marks in the right arm. Her serum creatinine level is 0.8 mg/dL. A chest x-ray reveals a right-sided opacity. Which of the following is the most appropriate immediate step in the management of this patient?
CorrectIncorrect -
Question 17 of 22
17. Question
A 48-year-old woman complains of persistent fatigue and daytime sleepiness. Her other medical problems include hypertension and rheumatoid arthritis. Her medications include naproxen, enalapril, and a multivitamin supplement with iron. She has no known drug allergies. Vital signs are within normal limits. Examination shows conjunctival pallor and bilateral hand joint deformities with tenderness to palpation at metacarpophalangeal joints. Stool testing for occult blood is negative.
Laboratory results are as follows:
Hemoglobin 8.2 g/dL Hematocrit 27% Mean corpuscular volume 84 fl Platelet 450,000/µL Leukocyte 6,000/µL Iron, serum 80 µg/dL Iron-binding capacity, serum 200 µg/dL (normal 250-370 µg/dL) Ferritin 300 ng/mL Serum erythropoietin 500 mU/mL (normal >100 mU/mL) Erythrocyte sedimentation rate 80 mm/h Which of the following would be the most appropriate next step in managing this patient’s anemia?
CorrectIncorrect -
Question 18 of 22
18. Question
A 56-year-old previously healthy man comes to the office with a 1-week history of worsening pain, paresthesia, and blackish discoloration of several fingers. He also has been experiencing fatigue and back pain over the past 2 months. He does not use tobacco, alcohol, or recreational drugs. Temperature is 37.2 C (99 F), blood pressure is 130/80 mm Hg, and pulse is 86/min and regular. The patient’s hand is shown in the image below:
There are areas of vertebral tenderness on palpation of the spine, and skin examination shows livedo reticularis and a retiform, purpuric lower extremity rash. Laboratory evaluation shows normocytic anemia and elevated levels of serum creatinine, serum calcium, and erythrocyte sedimentation rate. Serum total protein is elevated with reversal of the albumin/globulin ratio. Radiographs reveal scattered lytic vertebral lesions. Which of the following is most likely involved in the pathogenesis of this patient’s hand findings?
CorrectIncorrect -
Question 19 of 22
19. Question
A 68-year-old man with a long history of using chewing tobacco comes to the office for follow up of squamous cell carcinoma (SCC) of the oral cavity. A month ago, the patient was diagnosed with SCC of the left lateral tongue. He underwent resection, the surgical specimen showed clear margins, and he started additional treatment. The patient recently noticed a second ulcerative lesion on the right lateral tongue. Biopsy is performed and it also shows SCC. The second tumor most likely arose from which of the following mechanisms?
CorrectIncorrect -
Question 20 of 22
20. Question
A 78-year-old man comes to the office for the evaluation of persistent, severe, low back pain. The pain does not radiate down to his legs, and is not associated with bladder or bowel disturbances. He has metastatic prostate cancer, which has progressed despite hormonal therapy and chemotherapy. A bone scan done 6 weeks ago revealed the presence of 2 vertebral metastases in the L4 and L5 vertebral bodies. He was started on high-dose narcotic analgesics at that time. He says that the narcotics have “taken the edge” off the pain, but he continues to have severe debilitating pain. Neurological examination of the lower extremities is unremarkable. A repeat bone scan reveals results similar to the previous scan results. Which of the following is the next most appropriate step in controlling this patient’s symptoms?
CorrectIncorrect -
Question 21 of 22
21. Question
A 32-year-old woman comes to the postdischarge clinic after being admitted to the hospital 10 days ago for acute deep vein thrombosis. The patient was discharged after achieving a therapeutic INR (goal: 2-3) and has been taking warfarin every evening as prescribed. She reports persistent swelling and pain in her left leg for the past week. Medical history is significant for systemic lupus erythematosus with a positive lupus anticoagulant, as well as 2 spontaneous early pregnancy losses. Temperature is 37.2 C (99 F), blood pressure is 143/82 mm Hg, pulse is 86/min, and respirations are 22/min. Oxygen saturation is 95% on room air. BMI is 38 kg/m2. Fullness and tenderness of the left calf and inner thigh are present. Pedal pulses are full and symmetric. Point-of-care INR is 1.3. Lower-extremity compression sonography reveals an unchanged thrombus of the femoral vein. Which of the following is the best next step in managing this patient?
CorrectIncorrect -
Question 22 of 22
22. Question
A 64-year-old man admitted to a rehabilitation facility after right total knee replacement develops sudden-onset chest pain and shortness of breath on postoperative day 5. He has been receiving heparin for prophylaxis of deep vein thrombosis. Two months ago, he was hospitalized for pneumonia and improved with 1 week of antibiotics. Examination shows clear lungs and normal heart sounds. Cardiac enzymes are normal. ECG shows sinus tachycardia. CT angiogram reveals pulmonary embolism. Complete blood count shows platelets of 45,000/mm3. Prior to the knee surgery, his platelet count was 170,000/mm3. Heparin is discontinued and argatroban is started. Which of the following is the best approach for this patient?
CorrectIncorrect